LSAT and Law School Admissions Forum

Get expert LSAT preparation and law school admissions advice from PowerScore Test Preparation.

 Administrator
PowerScore Staff
  • PowerScore Staff
  • Posts: 8916
  • Joined: Feb 02, 2011
|
#74746
Complete Question Explanation

The correct answer choice is (A).

The stem here is referring to the last sentence of the passage. A more helpful way to rephrase the question would be, "What is the author doing structurally in the final sentence of the passage?" And as to that, a solid Pre-Phrase would be something along the lines of: the author is recommending a different course of behavior (specifically, tracking the development of factually accepted doctrines) for new historians of science. We then want to come down to the answer choices to find the answer choice that matches our Pre-Phrase.

Answer Choice (A): This is the correct answer. Prescription here is a synonym for "recommendation"; Answer Choice (A) gets at the idea from our Pre-Phrase that the author is recommending an alternate course of behavior for the new historians of science. As you pointed out, the author had not made a prior recommendation. The recommendation in the last sentence of the passage is not, then, alternate to the author's prior "prescription". Rather, it is an alternative to the current course of conduct of new historians of science (as laid out in the very first sentence of the passage).

Answer Choice (B). The author does not at any point present a paradox.

Answer Choice (C). I could see this answer tripping up some people, but a recommendation is certainly different than a prediction. The author is not predicting anything.

Answer Choice (D). I'm not sure the author exactly "concedes an argument" at any point during the passage. If she does, it's in the 3rd paragraph where (as you mentioned) she qualifies and tones down her stance from the 2nd paragraph. Regardless, that is in the penultimate paragraph. It is not what she is doing functionally in the last sentence of the passage. Is the thrust of the last sentence, "I admit..."? No, it's more along the lines of "Here's what the historians should be doing instead". That comports with Answer Choice (A).

Answer Choice (E). Similar to Answer Choice (C), the author is not anticipating anything here.
 yusrak
  • Posts: 22
  • Joined: Mar 19, 2020
|
#74704
My prephrase for this question was that the author admits that there is some truth to the new historian’s theory that ideology can influence scientific beliefs and then provides a recommendation for these new historians to further their research and analysis.

With this in mind I reviewed the answer choices with the following reasoning:
A) I thought this answer was wrong because I did not recall the author provided previous prescription/recommendation prior to the final paragraph. I assumed that “alters a prescription” suggests that the author must have provided a prescription previously in order to change it.
B) The author does not provide a paradox in the final paragraph.
C) The last paragraph does not mention anything about the future.
D) I chose this answer since the author qualified the strong rejection initially provided in the 2nd paragraph. I thought that the author qualified the rejection in both the 3rd and 4th paragraph.
E) There was no mention of future objections in the final paragraph.

Why is answer choice A right?

Best,
Yusra
 Paul Marsh
PowerScore Staff
  • PowerScore Staff
  • Posts: 290
  • Joined: Oct 15, 2019
|
#74727
Hi yusrak! The language of this question is a bit ambiguous (are we talking about the last couple paragraphs? just the last sentence?); I think we can blame that on the question being nearly 30 years old. A similar Reading Comprehension question today would likely be more specific.

In any case, please see the full explanation I posted above.

Hope that helps, feel free to follow up if you have any more specific questions.

Get the most out of your LSAT Prep Plus subscription.

Analyze and track your performance with our Testing and Analytics Package.